Last visit was: 20 Nov 2025, 08:08 It is currently 20 Nov 2025, 08:08
Close
GMAT Club Daily Prep
Thank you for using the timer - this advanced tool can estimate your performance and suggest more practice questions. We have subscribed you to Daily Prep Questions via email.

Customized
for You

we will pick new questions that match your level based on your Timer History

Track
Your Progress

every week, we’ll send you an estimated GMAT score based on your performance

Practice
Pays

we will pick new questions that match your level based on your Timer History
Not interested in getting valuable practice questions and articles delivered to your email? No problem, unsubscribe here.
Close
Request Expert Reply
Confirm Cancel
User avatar
RowdyHowdy
Joined: 28 Feb 2024
Last visit: 22 Oct 2025
Posts: 2
Own Kudos:
39
 [39]
Given Kudos: 36
Location: China
Posts: 2
Kudos: 39
 [39]
3
Kudos
Add Kudos
36
Bookmarks
Bookmark this Post
Most Helpful Reply
User avatar
MartyMurray
Joined: 11 Aug 2023
Last visit: 19 Nov 2025
Posts: 1,632
Own Kudos:
6,128
 [8]
Given Kudos: 173
GMAT 1: 800 Q51 V51
Expert
Expert reply
GMAT 1: 800 Q51 V51
Posts: 1,632
Kudos: 6,128
 [8]
7
Kudos
Add Kudos
1
Bookmarks
Bookmark this Post
General Discussion
User avatar
kakarot10
Joined: 04 Oct 2023
Last visit: 19 Mar 2025
Posts: 43
Own Kudos:
15
 [2]
Given Kudos: 28
Location: India
Concentration: Technology, Other
GMAT Focus 1: 595 Q79 V83 DI76
GPA: 3.5
GMAT Focus 1: 595 Q79 V83 DI76
Posts: 43
Kudos: 15
 [2]
2
Kudos
Add Kudos
Bookmarks
Bookmark this Post
User avatar
RowdyHowdy
Joined: 28 Feb 2024
Last visit: 22 Oct 2025
Posts: 2
Own Kudos:
Given Kudos: 36
Location: China
Posts: 2
Kudos: 39
Kudos
Add Kudos
Bookmarks
Bookmark this Post
kakarot10
It's a weaken question, and therefore the given conclusion of the passage is to be considered. The conclusion of the passage is that another region 1 did X and it resulted in Y, and thus we (in region 2) too should do X to get Y. However factors affecting budget deficit could be different in the 2 regions therefore answer is A . It weakens the arguments conclusion
­thanks! but how about B ? it also looks like a right answer.
User avatar
Fish181
Joined: 13 Dec 2023
Last visit: 22 Jan 2025
Posts: 135
Own Kudos:
1,003
 [2]
Given Kudos: 53
Status:Applying in R1 of 2024 to t15
Affiliations: University of Tennessee
Location: United States (CO)
Concentration: Strategy, Finance
GMAT Focus 1: 605 Q76 V84 DI80
GMAT Focus 2: 615 Q78 V86 DI78
GPA: 3.62
WE:Analyst (Consumer Packaged Goods)
GMAT Focus 2: 615 Q78 V86 DI78
Posts: 135
Kudos: 1,003
 [2]
2
Kudos
Add Kudos
Bookmarks
Bookmark this Post
A. It fails to adequately address the possibility that many factors affect provincial budget deficits.


- This says that you can't compare the two cities "apples to apples" so this is the correct answer because it's true.;

B. It confuses a claim about the budget deficit in one province with a similar claim about the deficit in another, neighboring province.

- it doesn't confuse the claim. It clearly mentions that there was a separate city north of them with the same problem that had a budget deficit. If you don't see this then you're not comprehending what the answer choice is telling you.

C. It overlooks the possibility that the budget deficit of the province immediately to the north was higher before that province allowed the union bargaining.

-It clearly states that the city north of them had a higher budget deficit before they enacted the law. So no.

D. It takes for granted that if the measure reduces the budget deficit, then it will not increase wages of government employees.

- That's not what's being asked here and is out of scope. Who cares? If it goes up or down it doesn't weaken the argument or strengthen it.

E. It fails to adequately distinguish a factual claim about the measure's likely consequences from a value judgment that the measure should be enacted.

- This one just tries to confuse you and hopes you pick it because it sounds confusing. I don't know what a value judgement is and I doubt you do either if you're reading this. But the part you should focus on is that it says "it fails to adequately distinguish a factual claim about the consequences." Well that just isn't true. It clearly states what it thinks will happen if it's enacted.
User avatar
whatsarc
Joined: 26 Aug 2023
Last visit: 19 Nov 2025
Posts: 109
Own Kudos:
Given Kudos: 78
Products:
Posts: 109
Kudos: 102
Kudos
Add Kudos
Bookmarks
Bookmark this Post
RowdyHowdy
­Editorial: Our province's legislature will soon vote on a measure that would allow government employee unions to bargain over wages. Some critics claim this measure would increase wages and thus exacerbate our province's budget deficit. But in fact, it would probably reduce the deficit. The province immediately north of ours allows such union bargaining, and its budget deficit is lower than our province's—and lower than it was before the province allowed union bargaining.

The editorial's argument is most vulnerable to criticism on which of the following grounds?

A. It fails to adequately address the possibility that many factors affect provincial budget deficits.

B. It confuses a claim about the budget deficit in one province with a similar claim about the deficit in another, neighboring province.

C. It overlooks the possibility that the budget deficit of the province immediately to the north was higher before that province allowed the union bargaining.

D.It takes for granted that if the measure reduces the budget deficit, then it will not increase wages of government employees.

E. It fails to adequately distinguish a factual claim about the measure's likely consequences from a value judgment that the measure should be enacted.

(source: GMAT focus online prep Exam1)

­
­Weird, I saw this question on third official test and you saw it on the first?

I thought the question pool were seperate for 1/2 and 3/4.
User avatar
Deepak23gmat
Joined: 20 Sep 2024
Last visit: 21 Dec 2024
Posts: 3
Given Kudos: 93
Location: India
GPA: 8.56
Posts: 3
Kudos: 0
Kudos
Add Kudos
Bookmarks
Bookmark this Post
KarishmaB Can you please help understand this.
User avatar
KarishmaB
Joined: 16 Oct 2010
Last visit: 20 Nov 2025
Posts: 16,267
Own Kudos:
Given Kudos: 482
Location: Pune, India
Expert
Expert reply
Active GMAT Club Expert! Tag them with @ followed by their username for a faster response.
Posts: 16,267
Kudos: 77,005
Kudos
Add Kudos
Bookmarks
Bookmark this Post
RowdyHowdy
­Editorial: Our province's legislature will soon vote on a measure that would allow government employee unions to bargain over wages. Some critics claim this measure would increase wages and thus exacerbate our province's budget deficit. But in fact, it would probably reduce the deficit. The province immediately north of ours allows such union bargaining, and its budget deficit is lower than our province's—and lower than it was before the province allowed union bargaining.

The editorial's argument is most vulnerable to criticism on which of the following grounds?

A. It fails to adequately address the possibility that many factors affect provincial budget deficits.

B. It confuses a claim about the budget deficit in one province with a similar claim about the deficit in another, neighboring province.

C. It overlooks the possibility that the budget deficit of the province immediately to the north was higher before that province allowed the union bargaining.

D.It takes for granted that if the measure reduces the budget deficit, then it will not increase wages of government employees.

E. It fails to adequately distinguish a factual claim about the measure's likely consequences from a value judgment that the measure should be enacted.

(source: GMAT focus online prep Exam1)

­

Premises:

Some critics claim that union bargaining would increase wages and thus exacerbate our province's budget deficit.
The province immediately north of ours allows such union bargaining, and its budget deficit is lower than our province's—and lower than it was before the province allowed union bargaining.

Conclusion: It would probably reduce the deficit.

The question stem asks us for a flaw in the reasoning. In these questions pre-thinking helps. What is the flaw in the logic here? Because the north province's deficit reduced after allowing union bargaining, the argument is assuming that the same will happen here. The very first issue with that is that we are not given that their deficit reduced because union bargaining reduced expense (and that other things stayed constant). The causality is not given. Their deficit could have reduced because of various other factors. We are not even given what impact union bargaining had on union's budget. It is possible that union bargaining even increased the deficit but other factors led to an overall decrease in the deficit.


A. It fails to adequately address the possibility that many factors affect provincial budget deficits.

Correct. It fails to address the possibility that many factors combine to give the budget deficit. Just because two things happened in sequence, it doesn't mean one led to the other.


B. It confuses a claim about the budget deficit in one province with a similar claim about the deficit in another, neighboring province.

The only claim given is "this measure would increase wages and thus exacerbate our province's budget deficit". No such claim about the deficit in another, neighboring province has been mentioned. The author only mentions what actually happened in another, neighboring province.


C. It overlooks the possibility that the budget deficit of the province immediately to the north was higher before that province allowed the union bargaining.

The author actually tells us that the budget deficit of the province immediately to the north was higher before that province allowed the union bargaining. It is not something he has overlooked


D.It takes for granted that if the measure reduces the budget deficit, then it will not increase wages of government employees.

The author does not talk about wages at all. He doesn't say whether the wages will increase or decrease etc.


E. It fails to adequately distinguish a factual claim about the measure's likely consequences from a value judgment that the measure should be enacted.

We do not know whether the claim given is "factual claim" (can be proven to be true or false). Hence we cannot say that the author's argument is flawed on this account. This is not the flaw that we can identify in this argument.

Answer (A)
User avatar
arushi118
Joined: 21 Jul 2024
Last visit: 20 Nov 2025
Posts: 52
Own Kudos:
Given Kudos: 404
Products:
Posts: 52
Kudos: 10
Kudos
Add Kudos
Bookmarks
Bookmark this Post
Why is B a wrong option for the question?
User avatar
egmat
User avatar
e-GMAT Representative
Joined: 02 Nov 2011
Last visit: 20 Nov 2025
Posts: 5,112
Own Kudos:
Given Kudos: 700
GMAT Date: 08-19-2020
Expert
Expert reply
Active GMAT Club Expert! Tag them with @ followed by their username for a faster response.
Posts: 5,112
Kudos: 32,889
Kudos
Add Kudos
Bookmarks
Bookmark this Post
Correct Answer: A
The editorial argues that allowing union bargaining will reduce the deficit because the northern province has union bargaining and a lower deficit. The flaw? It assumes union bargaining caused the lower deficit without considering other factors like economic growth, tax policy, or spending cuts.

Why Not B
"Confusing claims" means mixing up which province is which—like saying something about Province A but treating it as Province B. But the editorial is crystal clear throughout: "our province" stays ours, "the province north of us" stays theirs. There's no identity mix-up.

What the editorial actually does wrong:
It makes a lazy assumption: "Northern province has union bargaining + lower deficit → union bargaining must have caused it."
But what if their economy boomed? What if they raised taxes or cut spending? The editorial ignores all these possibilities. That's the real flaw—it's not confused about which province is which, it just fails to consider alternative explanations.

The key distinction:
  • Confusion (B) = mixing up identities
  • Incomplete reasoning (A) = ignoring alternative causes

Quick test: Does the author ever lose track of which province is which? No. So it's not B.


arushi118
Why is B a wrong option for the question?
Moderators:
GMAT Club Verbal Expert
7443 posts
GMAT Club Verbal Expert
231 posts
189 posts